Correction

Exercice 1 : Logique

1. Soient \(P\) et \(Q\) deux assertions. Montrer que les assertions "\(\text{non}(P \Longrightarrow Q)\)" et "\(P \text{ et } \text{non}(Q)\)" sont équivalentes.

Correction

L'assertion \(P\Longrightarrow Q\) est équivalente à "\(\text{non}(P) \text{ ou } Q\)". En effet :

On suppose que l'assertion \(P \Longrightarrow Q\) est vraie. Procédons par disjonction de cas. Si l'assertion \(P\) est fausse alors l'assertion "\(non(P) \text{ ou } Q\)" est vraie. De même si l'assertion \(P\) est vraie alors, d'après \(P \Longrightarrow Q\) vraie, l'assertion \(Q\) est vraie donc l'assertion "\(non(P) \text{ ou } Q\)" est également vraie.

Réciproquement on suppose que l'assertion "\(non(P) \text{ ou } Q\)" est vraie. On suppose l'assertion \(P\) vraie. Alors l'assertion \(non(P)\) est fausse et, comme l'assertion "\(non(P) \text{ ou } Q\)" est vraie, l'assertion \(P\) est vraie. Nous avons donc bien montré que l'assertion \(P\Longrightarrow Q\) est vraie.

Par conséquent par négation nous avons l'équivalence souhaitée.

2. Soient \(A,B,Q\) trois assertions telles que l'assertion \([A\Longrightarrow B] \Longrightarrow C\) est vraie. Qui est nécessaire à qui ? Qui est suffisant à qui ?

Correction
  • Si l'assertion \(A\) est vraie alors on ne peut conclure sur la valeur de l'assertion \(A \Longrightarrow B\) et donc on ne peut pas savoir si l'assertion \(B\) ou l'assertion \(C\) est vraie.

  • Si l'assertion \(A\) est fausse alors l'assertion \(A \Longrightarrow B\) est vraie car elle est équivalente à "\(\text{non}(A) \text{ et } B\)". Donc grâce à la vérité de l'assertion \([A\Longrightarrow B] \Longrightarrow C\) nous avons l'assertion \(C\) vraie. Par conséquent l'assertion \(\text{non}(A)\) est une condition suffisante à l'assertion \(C\), autrement dit l'assertion \(C\) est nécessaire à l'assertion \(\text{non}(A)\). Par contre nous ne pouvons pas conclure quant à la valeur de l'assertion \(B\).

  • Si l'assertion \(B\) est vraie alors l'assertion \(A \Longrightarrow B\) est vraie. Donc grâce à la vérité de l'assertion \([A\Longrightarrow B] \Longrightarrow C\) nous avons l'assertion \(C\) vraie. Par conséquent l'assertion \(B\) est une condition suffisante à l'assertion \(C\), autrement dit l'assertion \(C\) est nécessaire à l'assertion \(B\). Par contre nous ne pouvons pas conclure quant à la valeur de l'assertion \(A\).

  • Si l'assertion \(B\) est fausse alors on ne peut conclure sur la valeur de l'assertion \(A \Longrightarrow B\) et donc on ne peut pas savoir si l'assertion \(B\) ou l'assertion \(C\) est vraie.

  • Si l'assertion \(C\) est vraie alors on ne peut rien conclure.

  • Si l'assertion \(C\) est fausse alors, d'après la vérité de l'assertion \([A\Longrightarrow B] \Longrightarrow C\), nous avons l'assertion \(\text{non}(A\Longrightarrow B)\) vraie qui est équivalente à l'assertion "\(A \text{ et } \text{non}(B)\)". Donc les assertions \(A\) et \(\text{non}(B)\) sont vraies. Par conséquent l'assertion \(\text{non}(C)\) est une condition suffisante aux assertions \(A\) et \(\text{non}(B)\). Autrement dit les assertions \(A\) et \(\text{non}(B)\) sont des conditions nécessaires à l'assertion \(\text{non}(C)\).

En conclusion nous avons

\[\text{non}(A) \Longrightarrow C \quad \text{et} \quad B \Longrightarrow C.\]

Nous aurions également pu écrire

\[[[A\Longrightarrow B] \Longrightarrow C] \quad \Longleftrightarrow \quad [\text{non}(A \Longrightarrow B) \text{ ou } C] \quad \Longleftrightarrow \quad [[A \text{ et } \text{non}(B)] \text{ ou } C] \quad \Longleftrightarrow \quad [[A \text{ ou } C] \text{ et } [\text{non}(B) \text{ ou } C]].\]

Donc les assertions "\(A \text{ ou } C\)" et "\(\text{non}(B) \text{ ou } C\)" sont vraies et on conclut comme précédemment.

3. Soit \(f : \mathbb{R} \longrightarrow \mathbb{R}\). Que veulent signifier les assertions suivantes et les nier avec des quantificateurs.

  • \(\forall x\in \mathbb{R}, f(x) \neq 0\)

  • \(\forall M\in \mathbb{R}^*_+, \exists A \in \mathbb{R}_+^*, \forall x\geq A, f(x) > M\)

  • \(\forall x\in \mathbb{R}, f(x) > 0 \Longrightarrow x\leq 0\)

  • \(\forall \varepsilon \in \mathbb{R}_+^*, \exists \delta \in \mathbb{R}_+^*, \forall x,y\in I, |x-y| \leq \delta \Longrightarrow |f(x) - f(y)| \leq \varepsilon\)

Correction
  • La fonction \(f\) ne s'annule pas et la négation est
\[\exists x\in \mathbb{R}, \quad f(x) = 0.\]
  • La fonction \(f\) diverge vers \(+\infty\) en \(+\infty\) et la négation est
\[\exists M \in \mathbb{R}_+^*, \quad \forall A \in \mathbb{R}_+^*, \quad \exists x \geq A, \quad f(x) \leq M.\]
  • La fonction \(f\) est négative sur \(\mathbb{R}_+^*\) et la négation est
\[\exists x\in \mathbb{R}, \quad f(x) > 0 \quad \text{et} \quad x>0.\]
  • La fonction \(f\) est uniformément continue (\(\delta\) ne dépend pas ni \(x\) ni de \(y\)) et la négation est
\[\exists \varepsilon \in \mathbb{R}_+^*, \quad \forall \delta \in \mathbb{R}_+^*, \quad \exists x,y \in I, \quad |x-y| \leq \delta \quad \text{et} \quad |f(x) - f(y)| > \varepsilon.\]

Exercice 2 : Calcul algébrique

1. Calculer

\[\forall n\in \mathbb{N}^*, \quad \sum_{k=0}^n \dfrac{1}{(2k+1)(2k+3)}.\]
Correction

On considère la décomposition en éléments simples

\[\dfrac{1}{(2X+1)(2X+3)} = \dfrac{a}{2X+1} + \dfrac{b}{2X+3},\]

avec en multipliant par \(2X+1\) ou \(2X+3\) et en évaluant en \(-\dfrac{1}{2}\) ou en \(-\dfrac{3}{2}\)

\[a = \dfrac{1}{-2\dfrac{1}{2}+3} = \dfrac{1}{2} \quad \text{et} \quad b = \dfrac{1}{-2 \dfrac{3}{2} + 1} = - \dfrac{1}{2}.\]

Donc

\[\sum_{k=0}^n \dfrac{1}{(2k+1)(2k+3)} = \dfrac{1}{2} \sum_{k=0}^n \dfrac{1}{2k+1} - \dfrac{1}{2} \sum_{k=0}^n \dfrac{1}{2k+3} = \dfrac{1}{2} \sum_{k=0}^n \dfrac{1}{2k+1} - \dfrac{1}{2} \sum_{k=1}^{n+1} \dfrac{1}{2k+1} = \dfrac{1}{2}\left( 1 - \dfrac{1}{2n+3} \right).\]

2. Soit \(x \in \mathbb{R}\) et, pour tout \(n\in \mathbb{N}\),

\[P_n(x) = \prod_{k=0}^n (1+x^{2^k}).\]

Calculer \((1-x) P_n(x)\) et en déduire une expression simple de \(P_n(x)\).

Correction

Nous avons

\[(1-x)P_0(x) = (1-x)(1+x) = 1-x^2, \quad (1-x)P_1(x) = (1-x)(1+x)(1+x^2) = (1-x^2)(1+x^2) = 1-x^4, \quad (1-x)P_2(x) = P_1(x) (1+x^4) = 1-x^8.\]

Ainsi nous pouvons montrer par récurrence que

\[(1-x) P_n(x) = 1-x^{2^{n+1}}.\]
  • Pour \(n = 0\) nous avons bien l'initialisation.

  • On suppose le résultat vrai au rang \(n\). Alors

\[(1-x)P_{n+1}(x) = (1-x)P_n(x) (1+x^{2^{n+1}}) = (1-x^{2^{n+1}})(1+x^{2^{n+1}}) = 1-x^{2^{n+2}}.\]

Le principe de récurrence permet de conclure. Donc, pour \(x\neq 1\),

\[P_n(x) = \dfrac{1-x^{2^{n+1}}}{1-x}\]

avec

\[\lim_{x\to 1} \dfrac{1-x^{2^{n+1}}}{1-x} = \lim_{x\to 1} \sum_{k=0}^{2^n} x^k = 2^{n+1} = P_n(1).\]

Donc la formule est également valable en \(x = 1\) au sens prolongement par continuité.

3. Calculer les sommes suivantes pour \(n \in \mathbb{N}\) :

\[A = \sum_{k=0}^n k \binom{n}{k}, \quad B = \sum_{k=0}^n (-1)^k k \binom{n}{k} \quad \text{et} \quad C = \sum_{k=0}^n k^2 \binom{n}{k}.\]
Correction
  • Nous avons pour \(n\geq 1\) et \(k \in \{1, ..., n\}\)
\[k\binom{n}{k} = k \dfrac{n!}{k!(n-k)!} = n \dfrac{(n-1)!}{(k-1)!(n-1-(k-1))!} = n \binom{n-1}{k-1}.\]

Donc

\[A = 0 + n \sum_{k=1}^n \binom{n-1}{k-1} = n \sum_{\ell = 0}^{n-1} \binom{n-1}{\ell} = n 2^{n-1}.\]

Et pour \(n = 0\) nous avons \(A = 0\).

  • Nous avons de même pour \(n\geq 1\)
\[B = 0 + n \sum_{k=1}^n (-1)^k \binom{n-1}{k-1} = - n \sum_{\ell = 0}^{n-1} (-1)^\ell \binom{n-1}{\ell} = -n(-1 + 1)^{n-1} = -n 0^{n-1}.\]

Donc pour \(n = 1\) nous avons \(B = -1\) et pour \(n\geq 2\) nous avons \(B = 0\). Puis pour \(n = 0\) nous avons \(B = 0\).

  • Nous avons pour tout \(n\in \{2, ...,\}\) et \(k \in \{2, ..., n\}\)
\[k^2 \binom{n}{k} = k(k-1) \binom{n}{k} + k \binom{n}{k} = n(n-1)\binom{n-2}{k-2} + n \binom{n-1}{k-1}.\]

Donc

\[C = 0 + 1 \binom{n}{1} + n(n-1) \sum_{k=2}^n \binom{n-2}{k-2} + n \sum_{k=2}^n \binom{n-1}{k-1} = n + n(n-1) \sum_{\ell = 0}^{n-2} \binom{n-2}{\ell} + n \sum_{\ell = 0}^{n-1} \binom{n-1}{\ell} = n + n(n-1)2^{n-2} + n 2^{n-1}.\]

4. Soit \(x\in \mathbb{R}\) et \(n\in \mathbb{N}^*\). On considère

\[U = \sum_{k=0}^n \binom{n}{k} \cos(kx) \quad \text{et} \quad V = \sum_{k=0}^n \binom{n}{k} \sin(kx).\]

Déterminer des expressions simplifiées de \(U\) et \(V\).

Correction

Nous avons

\[U + iV = \sum{k=0}^n \binom{n}{k}(\cos(kx) + i\sin(kx)) = \sum_{k=0}^n \binom{n}{k} (e^{ix})^k = (1+e^{ix})^n = e^{i \frac{nx}{2}}\left( e^{-i \frac{x}{2}} + e^{i \frac{x}{2}} \right)^n = 2^n e^{i \frac{nx}{2}} \left( \cos \left( \dfrac{x}{2} \right) \right)^n.\]

Donc

\[U = 2^n \cos\left( \dfrac{nx}{2} \right) \left( \cos \left( \dfrac{x}{2} \right) \right)^n \quad \text{et} \quad V = 2^n \sin\left( \dfrac{nx}{2} \right) \left( \cos \left( \dfrac{x}{2} \right) \right)^n.\]

5. Soit \(n\in \mathbb{N}\) tel que \(n \geq 2\). Calculer

\[\sum_{1\leq i<j\leq n} |i-j| \quad \text{et} \quad \sum_{1\leq i,j\leq n} |i-j|.\]
Correction

Nous avons

\[\sum_{1\leq i<j\leq n} |i-j| = \sum_{1\leq i<j\leq n} (j-i) = \sum_{j=1}^n j \sum_{i=1}^{j-1} 1 - \sum_{i=1}^n i \sum_{j=i+1}^n 1 = \sum_{j=1}^n (j^2 - j) - \sum_{i=1}^n (ni - i^2) = \dfrac{n(n+1)(2n+1)}{6} - \dfrac{n(n+1)}{2} - \dfrac{n^2(n+1)}{2} + \dfrac{n(n+1)(2n+1)}{6} = \dfrac{2n(n+1)(2n+1) -3n(n+1) - 3n^2(n+1)}{3} = \dfrac{n(n+1)(4n+2 - 3-3n)}{6} = \dfrac{n(n+1)(n-1)}{6}.\]

Et par symétrie des rôles entre \(i\) et \(j\) dans le calcul précédent

\[\sum_{1\leq i,j\leq n} |i-j| = \sum_{1\leq i<j\leq n} |i-j| + \sum_{1\leq j<i\leq n} |i-j| = \dfrac{n(n+1)(n-1)}{3}.\]

Exercice 3 : Complexes

1. Soit \(n\in \mathbb{N}\) tel que \(n\geq 2\).

1.a. Montrer que \(\mathbb{U}_n\) l'ensemble des racines \(n\)-ièmes de l'unité est un groupe abélien.

Correction

Montrons que l'ensemble \(\mathbb{U}_n\) est un sous-groupe du groupe abélien \((\mathbb{C}^*, \times)\).

  • \(1^n = 1\) donc \(1 \in \mathbb{U}_n\).

  • Soit \(x,y \in \mathbb{U}_n\). Alors \((xy^{-1})^n = x^n (y^n)^{-1} = 1\) donc \(xy^{-1} \in \mathbb{U}_n\).

Par conséquent nous avons montré que l'ensemble \(\mathbb{U}_n\) est un groupe abélien pour la loi \(\times\).

1.b. Déterminer les éléments de \(\mathbb{U}_n\).

Correction

Soit \(x\in \mathbb{U}_n\). Donc \(x^n = 1\). En particulier \(|x|^n = 1\) d'où \(|x| = 1\). Ainsi il existe \(\theta \in \mathbb{R}\) tel que \(x = e^{i\theta}\). Donc \(1 = x^n = e^{in\theta}\). Donc il existe \(k\in \mathbb{Z}\) tel que \(n\theta = 2k\pi\) i.e. \(\theta = \dfrac{2k\pi}{n}\). Réciproquement \(e^{i \frac{2k\pi}{n}} \in \mathbb{U}_n\) pour tout \(k\in \mathbb{Z}\). Par conséquent

\[\mathbb{U}_n = \left\{ e^{i\frac{2k\pi}{n}}, \quad k\in \mathbb{Z} \right\} = \left\{ e^{i\frac{2k\pi}{n}}, \quad k\in \{0, ..., n-1\} \right\}\]

la dernière égalité étant justifiée par le fait que la suite \(\left( e^{i\frac{2k\pi}{n}} )_{n\in \mathbb{Z}}\) est \(n\)-périodique.

1.c. Montrer que leur somme est nulle.

Correction

Nous avons

\[\sum_{k=0}^{n-1} e^{i\frac{2k\pi}{n}} = \dfrac{1-e^{i\frac{2n\pi}{n}}}{1-e^{i\frac{2\pi}{n}}} = 0.\]

2. On considère les complexes suivants

\[u = e^{i \frac{2\pi}{11}}, \quad S = u + u^3 + u^4 + u^5 + u^9 \quad \text{et} \quad T = u^2 + u^6 + u^7 + u^8 + u^{10}.\]

2.a. Montrer que les complexes \(S\) et \(T\) sont conjugués.

Correction

Nous avons

\[\overline{S} = \overline{u} + \overline{u}^3 + \overline{u}^4 + \overline{u}^5 + \overline{u}^9 = e^{-i \frac{2\pi}{11}} + e^{-i\frac{6\pi}{11}} + e^{-i\frac{8\pi}{11}} + e^{-i\frac{10\pi}{11}} + e^{-i\frac{18\pi}{11}}\]

et

\[T = e^{i \frac{4\pi}{11}} + e^{i\frac{12\pi}{11}} + e^{i\frac{14\pi}{11}} + e^{i\frac{16\pi}{11}} + e^{i\frac{20\pi}{11}} = e^{-i\frac{18\pi}{11}} + e^{-i\frac{10\pi}{11}} + e^{-i\frac{8\pi}{11}} + e^{-i\frac{6\pi}{11}} + e^{-i \frac{2\pi}{11}}.\]

Donc \(\overline{S} = T\).

2.b. Montrer que la partie imaginaire du complexe \(S\) est positive.

Correction

Nous avons

\[\text{Im}(S) = \text{Im}(u) + \text{Im}(u^3) + \text{Im}(u^4) + \text{Im}(u^5) + \text{Im}(u^9) = \sin\left(\dfrac{2\pi}{11}\right) + \sin \left( \dfrac{6\pi}{11} \right) + \sin\left( \dfrac{8\pi}{11} \right) + \sin\left( \dfrac{10 \pi}{11} \right) + \sin\left( \dfrac{18\pi}{11} \right),\]

avec

\[\sin\left(\dfrac{2\pi}{11}\right), \sin\left( \dfrac{8\pi}{11} \right), \sin\left( \dfrac{10 \pi}{11} \right) > 0\]

car

\[\dfrac{2\pi}{11}, \dfrac{8\pi}{11}, \dfrac{10 \pi}{11} \in ~]0,\pi[,\]

et

\[\in\left( \dfrac{6\pi}{11} \right) + \sin\left( \dfrac{18\pi}{11} \right) = 2 \cos\left( \dfrac{6\pi}{11} \right) \sin \left( \dfrac{12\pi}{11} \right) > 0\]

car

\[\dfrac{6\pi}{11} \in ~\left]\dfrac{\pi}{2}, \dfrac{3\pi}{2}\right[ \quad \text{et} \quad \dfrac{12\pi}{11} \in ~]\pi, 2\pi[.\]

Par conséquent \(\text{Im}(S) > 0\).

2.c. Montrer que \(S+T = -1\) et \(S\times T = 3\). En déduire les valeurs de \(S\) et \(T\).

Correction

Nous avons

\[S + T = \sum_{k=1}^{10} u^k = \dfrac{u-u^{11}}{1-u} = \dfrac{u-1}{1-u} = -1.\]

et, en développant le produit

\[S\times T = ... = 5+2(S+T) = 3.\]

Par conséquent les complexes \(S\) et \(T\) sont les racines du polynôme

\[P = X^2 - (S+T) X + ST = X^2 + X + 3\]

de discriminant \(\Delta = -11\) donc de racines

\[x_1 = \dfrac{-1+i\sqrt{11}}{2} \quad \text{et} \quad x_2 = \dfrac{-1+i\sqrt{11}}{2}.\]

Or \(\text{Im}(S) > 0\) d'après la question 2.b. donc

\[S = x_2 = \dfrac{-1+i\sqrt{11}}{2} \quad \text{et} \quad T = x_1 = \dfrac{-1-i\sqrt{11}}{2}\]

2.d. Montrer que

\[i \tan \left( \dfrac{3\pi}{11} \right) = \dfrac{u^3 - 1}{u^3 + 1} = - \sum_{k=1}^{10} (-u^3)^k.\]

Puis que

\[4i \sin \left( \dfrac{2\pi}{11} \right) = 2 (u - u^{10}).\]
Correction

Nous avons directement

\[\sum_{k=1}^{10} (-u^3)^k = \dfrac{-u^3 + (u^3)^{11}}{1+u^3} = \dfrac{1-u^3}{1+u^3}\]

et également

\[i \tan \left( \dfrac{3\pi}{11} \right) = \dfrac{i\sin\left( \frac{3\pi}{11} \right)}{\cos\left( \frac{3\pi}{11} \right)} = \dfrac{e^{i\frac{3\pi}{11}} - e^{-i\frac{3\pi}{11}}}{e^{i\frac{3\pi}{11}} + e^{-i\frac{3\pi}{11}}} = \dfrac{e^{i\frac{6\pi}{11}} - 1}{e^{i\frac{6\pi}{11}} + 1} = \dfrac{u^3 - 1}{u^3 + 1}.\]

Puis

\[4i \sin \left( \dfrac{2\pi}{11} \right) = 2 (e^{i \frac{2\pi}{11}} - e^{- i \frac{2\pi}{11}}) = 2 (e^{i \frac{2\pi}{11}} - e^{ i \frac{20\pi}{11}}) = 2(u - u^{10}).\]

2.e. En déduire que

\[\tan\left( \dfrac{3\pi}{11} \right) + 4 \sin\left( \dfrac{2\pi}{11} \right) = i(T-S) = \sqrt{11}.\]
Correction

Nous avons d'après la question précédente

\[i\tan\left( \dfrac{3\pi}{11} \right) + 4 i \sin\left( \dfrac{2\pi}{11} \right) = - \sum_{k=1}^{10} (-u^3)^k + 2(u - u^{10}) = u^3 - u^6 + u^9 - u + u^4 - u^7 + u^{10} - u^2 + u^5 - u^8 + 2u - 2^{10} = u^3 - u^6 + u^9 + u + u^4 - u^7 - u^{10} - u^2 + u^5 - u^8 = S - T = i \sqrt{11}.\]

Donc

\[\tan\left( \dfrac{3\pi}{11} \right) + 4 \sin\left( \dfrac{2\pi}{11} \right) = i(T-S) = \sqrt{11}.\]

Exercice 4 : Systèmes linéaires

1. Résoudre, en fonction de \(m\in \mathbb{R}\), le système

\[\begin{cases} mx & + & y & = & 2 \\ (m^2 + 1) x & + & 2my & = & 1. \end{cases}\]
Correction

On considère \((x,y) \in \mathbb{R}^2\) solution du système. Alors

\[y = 2 - mx.\]

Donc

\[1 = (m^2+1) x + 2my = (m^2 + 1) x + 4m - 2m^2x = (1-m^2) x + 4m.\]

Donc si \(m\notin \{1, -1\}\) alors

\[x = \dfrac{1-4m}{1-m^2} \quad \text{et} \quad y = 2 -mx = \dfrac{2 - 2m^2 - m + 4m^2}{1-m^2} = \dfrac{2m^2 - m - 2}{1-m^2}.\]

Puis si \(m = 1\) alors

\[\begin{cases} x & + & y & = & 2 \\ 2x & + & 2y & = & 1 \end{cases}\]

ce qui ne peut pas. Donc danc ce cas le système n'admet pas de solutions.

Enfin si \(m = -1\) alors

\[\begin{cases} -x & + & y & = & 2 \\ 2 x & - & 2y & = & 1. \end{cases}\]

ce qui ne peut pas non plus. Donc dans ce cas également le système n'admet pas de solutions.

2. Résoudre, en fonction de \(a,b,c \in \mathbb{R}\), le système linéaire

\[\begin{cases} x & + & 2y & - & 3z & = & a \\ 2x & + & 6y & - & 11z & = & b \\ x & - & 2y & + & 6z & = & c. \end{cases}\]
Correction

On effectue l'algorithme du pivot de Gauss. Le premier pivot est non nul \((1\neq 0)\). On peut donc effectuer les opérations élémentaires suivantes

\[L_2 \longleftarrow L_2 - 2L_1 \quad \text{et} \quad L_3 \longleftarrow L_3 - L_1\]

pour obtenir le système équivalent

\[\begin{cases} x & + & 2y & - & 3z & = & a \\ & & 2y & - & 5z & = & b - 2a \\ & - & 4y & + & 9z & = & c - a. \end{cases}\]

Le second pivot est également non nul \((2\neq 0)\). On peut donc effectuer l'opération élémentaire suivante

\[L_3 \longleftarrow L_3 + 2 L_2\]

pour obtenir le système équivalent

\[\begin{cases} x & + & 2y & - & 3z & = & a \\ & & 2y & - & 5z & = & b - 2a \\ & & & - & z & = & c - 5a + 2b. \end{cases}\]

Puis par méthode de rémontée nous obtenons

\[z = 5a - 2b - c, \quad y = \dfrac{b}{2} - a + \dfrac{5}{2} z = \dfrac{23}{2}a - \dfrac{9}{2}b - \dfrac{5}{2}c \quad \text{et} \quad x = a - 2y + 3z = -7a +3b + 2c.\]

Nous pouvons également remarquer que nous avons obtenons la matrice inverse \(A^{-1}\) de la matrice \(A\) associée au système :

\[A^{-1} = \left( \begin{array}{ccc} -7 & 3 & 2 \\ \frac{23}{2} & - \frac{9}{2} & - \frac{5}{2} \\ 5 & -2 & -1 \end{array} \right).\]

Exercice 5 : Ensembles et applications

On considère deux ensembles \(X\) et \(Y\) et une application \(f : X \longrightarrow Y\).

1. Montrer que l'application \(f\) est injective si et seulement si pour tout ensemble \(Z\) et toutes applications \(g,h : Z \longrightarrow X\)

\[f\circ g = f\circ h \quad \Longrightarrow \quad g = h.\]
Correction
  • On suppose que l'application \(f\) est injective. Soit \(g,h : Z \longrightarrow X\) telles que \(f\circ g = f\circ h\). Soit \(z\in Z\). Alors \(f(g(z)) = f(h(z))\). Donc par injectivité de l'application \(f\) nous obtenons \(g(z) = h(z)\). Donc \(h = g\).

  • Réciproquement on suppose que

\[\forall g,h : Z\longrightarrow X, \quad f\circ g = f\circ h \quad \Longrightarrow \quad g = h.\]

Soit \(x,y\in X\) tels que \(f(x) = f(y)\). On considère l'ensemble \(Z = \{x,y\}\) et les applications \(g,h : Z\longrightarrow X\) définies par

\[g(x) = g(y) = x \quad \text{et} \quad h(x) = h(y) = y.\]

Alors

\[f(g(y)) = f(g(x)) = f(x) = f(y) = f(h(x)) = f(h(y)).\]

Donc \(f \circ g = f\circ h\), d'où, par hypothèse, \(g = h\). Ainsi

\[x = g(x) = h(x) = y.\]

Par conséquent l'application \(f\) est injective.

2. Montrer que l'application \(f\) est surjective si et seulement si pour tout ensemble \(Z\) et toutes applications \(g,h : Y \longrightarrow Z\)

\[g\circ f = h\circ f \quad \Longrightarrow \quad g = h.\]
Correction
  • On suppose que l'application \(f\) est surjective. Soit \(g,h : Y \longrightarrow Z\) telles que \(g\circ f = h\circ f\). Soit \(y\in Y\). Alors, par surjectivité de l'application \(f\), il existe \(x\in X\) tel que \(y = f(x)\). Donc
\[g(y) = g(f(x)) = h(f(x)) = h(y).\]

Ainsi \(g = h\).

  • Pour montrer le sens réciproque nous allons procéder par contraposée. On suppose que l'application \(f\) n'est pas surjective : il existe \(y_0 \in Y\) tel que
\[\forall x\in X, \quad y_0\neq f(x).\]

On considère alors les applications \(g,h : Y \longrightarrow Y\cup\{0,1\}\) définies par

\[\forall y \in Y\backslash \{y_0\}, \quad g(y) = h(y) = y, \quad g(y_0) = 0, \quad h(y_0) = 1.\]

Alors pour tout \(x \in X, f(x) \in Y\backslash \{y_0\}\) donc

\[g(f(x)) = f(x) = h(f(x)).\]

Donc \(g\circ f = h\circ f\) mais \(g\neq h\) parce que \(g(y_0) = 0 \neq 1 = h(y_0)\). Nous avons donc bien montré par contraposée l'implication réciproque.

Exercice 6 : Fonctions usuelles

1. Montrer que la fonction \(\text{sh}\) est une bijection de \(\mathbb{R}\) sur un intervalle \(I\) à préciser. On note \(\text{argsh}\) sa bijection réciproque.

Correction

La fonction \(\text{sh}\) est dérivable de fonction dérivée \(\text{sh}' = \text{ch} > 0\) donc la fonction \(\text{sh}\) est strictement croissante donc injective. De plus \(\lim_{+\infty} \text{sh} = +\infty\) et \(\lim_{-\infty} \text{sh} = -\infty\) donc, par théorème des valeurs intermédiaires, \(\text{Im}(\text{sh}) = \mathbb{R}\). Par conséquent la fonction \(\text{sh}\) est une bijection de \(\mathbb{R}\) dans \(\mathbb{R}\).

2. La fonction \(\text{argsh}\) est-elle dérivable sur \(I\) ? Lorsque c'est possible, calculer les dérivées \(\text{argsh}'(x)\).

Correction

Comme \(\text{sh}' = \text{ch} > 0\) la fonction \(\text{argsh}\) est dérivable sur \(\mathbb{R}\) et

\[\forall x\in \mathbb{R}, \quad \text{argsh}^{-1}(x) = \dfrac{1}{\text{ch}(\text{argsh}(x))} = \dfrac{1}{\sqrt{1+ (\text{sh}(\text{argsh}(x)))^2}} = \dfrac{1}{\sqrt{1+x^2}}.\]

3. On considère la fonction \(\phi\) définie par

\[\phi(x) = \ln(x+\sqrt{x^2+1}).\]

3.a. Déterminer l'ensemble de définition de la fonction \(\phi\). Etudier sa parité. Etudier sa dérivabilité et calculer sa dérivée lorsqu'elle est définie. En déduire que

\[\phi = \text{argsh}.\]
Correction
  • Pour tout \(x\in \mathbb{R}\) nous avons
\[\sqrt{x^2 + 1} > \sqrt{x^2} = |x| > -x.\]

Donc

\[x+\sqrt{x^2 + 1} > 0.\]

Ainsi la fonction \(\phi\) est définie sur \(\mathbb{R}\).

  • Pour tout \(x\in \mathbb{R}\) nous avons
\[\phi(x) + \phi(-x) = \ln(x+\sqrt{x^2 + 1}) + \ln(-x+\sqrt{x^2+1}) = \ln\left( (x+\sqrt{x^2+1})(-x+\sqrt{x^2+1}) \right) = \ln(x^2 + 1 - x^2) = 0.\]

Donc la fonction \(\phi\) est impaire sur \(\mathbb{R}\).

  • La fonction \(\phi\) est dérivable sur \(\mathbb{R}\) comme composée et somme de telles fonctions et
\[\forall x\in \mathbb{R}, \quad \phi'(x) = \dfrac{1 + \dfrac{x}{\sqrt{x^2+1}}}{x+\sqrt{x^2+1}} = \dfrac{1}{\sqrt{x^2+1}}.\]
  • Ainsi il existe \(c \in \mathbb{R}\) tel que
\[\forall x \in \mathbb{R}, \quad \phi(x) = \text{argsh}(x) + c.\]

Or \(\phi(0) = 0 = \text{argsh}(0)\), donc \(c = 0\) et \(\phi = \text{argsh}\).

3.b. Retrouver par un calcul direct (sans passer par les dérivées) le résultat précédent.

Correction

Pour tout \(x\in \mathbb{R}\) nous avons

\[\text{sh}(\phi(x)) = \dfrac{e^{\phi(x)} - e^{-\phi(x)}}{2} = \dfrac{x+\sqrt{x^2+1}-\dfrac{1}{x+\sqrt{x^2+1}}}{2} = \dfrac{(x+\sqrt{x^2+1})^2 - 1}{2(x+\sqrt{x^2+1})} = \dfrac{2x^2 + 2x\sqrt{x^2+1}}{2(x+\sqrt{x^2+1})} = \dfrac{x^2 + x\sqrt{x^2+1}}{x+\sqrt{x^2+1}} = x.\]

Donc

\[\phi = \text{sh}^{-1} = \text{argsh}.\]

Exercice 7 : Primitives et intégrales

1. Déterminer une primitive de la fonction \(x \longmapsto x^2 \arctan(x)\) sur \(\mathbb{R}\).

Correction

Une primitive de cette fonction \(f : x\longmapsto x^2 \arctan(x)\) est la fonction \(F : \mathbb{R} \longrightarrow \mathbb{R}\) définie par

\[\forall x\in \mathbb{R}, \quad F(x) = \int_0^x t^2 \arctan(t) dt.\]

Or, par intégration par parties, nous avons

\[\forall x\in \mathbb{R}, \quad F(x) = \left[ \dfrac{t^3}{3} \arctan(t) \right]_0^x - \int_0^x \dfrac{t^3}{3} \dfrac{1}{1+t^2} dt = \dfrac{1}{3} x^3 \arctan(x) - \dfrac{1}{3} \int_0^x \dfrac{t^3}{1+t^2} dt,\]

avec, par division euclidienne,

\[\dfrac{X^3}{1+X^2} = X - \dfrac{X}{1+X^2}.\]

Donc

\[\forall x\in \mathbb{R}, \quad F(x) = \dfrac{1}{3}x^3 \arctan(x) - \dfrac{1}{3}\int_0^x tdt + \dfrac{1}{3} \int_0^x \dfrac{t}{1+t^2} dt = \dfrac{1}{3}x^3 \arctan(x) - \dfrac{1}{6} x^2 + \dfrac{1}{6} \ln(1+x^2).\]

2. Calculer l'intégrale

\[\int_0^{\ln(2)} \dfrac{dt}{1+\text{th}(t)}.\]
Correction

Nous avons par changement de variable \(u = \text{th}(t), du = (1-\text{th}(t))^2 dt = (1-u^2) dt\)

\[\int_0^{\ln(2)} \dfrac{dt}{1+\text{th}(t)} = \int_0^{\text{th}(\ln(2))} \dfrac{du}{(1-u^2)(1+u)} = \int_0^{\frac{3}{5}} \dfrac{du}{(1-u)(1+u)^2}.\]

Or, par décomposition en éléments simples, nous avons

\[\dfrac{1}{(1-X)(1+X)^2} = \dfrac{a}{1-X} + \dfrac{b}{1+X} + \dfrac{c}{(1+X)^2}\]

avec de manière classique

\[a = \dfrac{1}{(1+1)^2} = \dfrac{1}{4},\]
\[c = \dfrac{1}{1-(-1)} = \dfrac{1}{2}\]

et pour calculer \(b\) on multiplie par \((1+X)^2\) puis on dérive formellement avant d'évaluer en \(-1\) :

\[\dfrac{1}{1-X} = (1+X)^2 Q a + (1+X) b + c, \quad Q = \dfrac{1}{1-X},\]

d'où

\[\dfrac{1}{(1-X)^2} = 2(1+X) Q a + (1+X)^2 Q' a + b\]

et

\[b = \dfrac{1}{(1-(-1))^2} = \dfrac{1}{4}.\]

Ainsi

\[\int_0^{\ln(2)} \dfrac{dt}{1+\text{th}(t)} = \dfrac{1}{4} \int_0^{\frac{3}{5}} \dfrac{du}{1-u} + \dfrac{1}{4} \int_0^{\frac{3}{5}} \dfrac{du}{1+u} + \dfrac{1}{2} \int_0^{\frac{3}{5}} \dfrac{du}{(1+u)^2} = \dfrac{1}{4} \left[ -\ln(1-u) \right]_0^{\frac{3}{5}} + \dfrac{1}{4} \left[ \ln(1+u) \right]_0^{\frac{3}{5}} + \dfrac{1}{2} \left[ -\dfrac{1}{1+u} \right]_0^{\frac{3}{5}} = -\dfrac{1}{4} \ln\left( \dfrac{2}{5} \right) + \dfrac{1}{4} \ln\left( \dfrac{8}{5} \right) + \dfrac{1}{2} \dfrac{3}{8} = \dfrac{1}{4} \ln \left( \dfrac{16}{25} \right) + \dfrac{3}{16} = \dfrac{1}{2} \ln\left( \dfrac{4}{5} \right) + \dfrac{3}{16}.\]

3. On considère pour tout \(p\in \mathbb{N}\) l'intégrale

\[I_p = \int_0^1 \dfrac{x^p}{1+x^2} dx.\]

Calculer \(I_0\) et \(I_1\). Déterminer \(I_p + I_{p+2}\) en fonction de \(p\). En déduire \(I_2\) et \(I_3\).

Correction

Nous avons

\[I_0 = \int_0^1 \dfrac{1}{1+x^2} dx = \arctan(1) = \dfrac{\pi}{4}\]

et

\[I_1 = \int_0^1 \dfrac{x}{1+x^2} dx = \dfrac{1}{2} \left[ \ln(1+x^2) \right]_0^1 = \dfrac{\ln(2)}{2}.\]

Puis pour \(p \in \mathbb{N}\) nous avons

\[I_p + I_{p+2} = \int_0^1 \dfrac{x^p + x^{p+2}}{1+x^2} dx = \int_0^1 x^p dx = \dfrac{1}{p+1}.\]

Ainsi

\[I_2 = \dfrac{1}{0+1} - I_0 = \dfrac{3\pi}{4}\]

et

\[I_3 = \dfrac{1}{1+1} - I_1 = \dfrac{1-\ln(2)}{2}.\]

4. On considère pour tout \(p,q\in \mathbb{N}\) l'intégrale

\[J_{p,q} = \int_0^1 \dfrac{x^p}{(1+x^2)^q} dx.\]

4.a. Calculer \(J_{1,q}\) en fonction de \(q\).

Correction

Soit \(q\in \mathbb{N}\).

  • Si \(q = 0\) alors \(J_{1,0} = \int_0^1 x dx = \dfrac{1}{2}\).

  • Si \(q = 1\) alors \(J_{1,1} = \int_0^1 \dfrac{x}{1+x^2} dx = \dfrac{1}{2} \ln(2)\).

\[J_{1,q} = \int_0^1 \dfrac{x}{(1+x^2)^q} dx = \int_0^1 x (1+x^2)^{-q} dx = \dfrac{1}{2} \left[ \dfrac{(1+x^2)^{-q+1}}{-q+1} \right]_0^1 = \dfrac{1}{2(1-q)} \left( \dfrac{1}{2^{q-1}} - 1 \right) = \dfrac{1 - 2^{q-1}}{2^q(1-q)}.\]

4.b. Déterminer une relation entre \(J_{0,q+1}\) et \(J_{0,q}\).

Correction

Soit \(q\in \mathbb{N}\). Alors par intégration par parties

\[J_{0,q} = \int_0^1 \dfrac{1}{(1+x^2)^q} \times 1 dx = \left[ \dfrac{1}{(1+x^2)^q} x \right]_0^1 + 2q \int_0^1 \dfrac{x}{(1+x^2)^{q+1}} x dx = \dfrac{1}{2^q} + 2q \int_0^1 \dfrac{x^2+1}{(1+x^2)^{q+1}} dx - 2q \int_0^1 \dfrac{1}{(1+x^2)^{q+1}} dx = \dfrac{1}{2^q} + 2q J_{0,q} - 2q J_{0,q+1}.\]

Donc

\[J_{0,q+1} = \dfrac{1}{2^{q+1} q} + \dfrac{2q-1}{2q} J_{0,q}.\]

4.c. Déterminer \(\alpha, \beta \in \mathbb{R}\) tels que

\[\forall p\geq 2, q\geq 1, \quad J_{p,q} = \alpha J_{p-2,q-1} + \beta J_{p-2,q}.\]
Correction

Soient \(p \geq 2\) et \(q\geq 2\). Alors

\[J_{p,q} = \int_0^1 \dfrac{x^p}{(1+x^2)^q} dx = \int_0^1 \dfrac{x^{p-2}(1+x^2)}{(1+x^2)^q} dx - \int_0^1 \dfrac{x^{p-2}}{(1+x^2)^q} dx = J_{p-2,q-1} - J_{p-2,q}.\]

Exercice 8 : Equations différentielles

1.a. Déterminer une équation différentielle linéaire du premier ordre dont les solutions sur \(\mathbb{R}_+^*\) sont les fonctions

\[x\longmapsto \arctan(x) - \dfrac{1}{2x} \ln(x^2+1) + \dfrac{\lambda}{x}, \quad \lambda \in \mathbb{R}.\]
Correction

Le terme avec \(\lambda\) est associé à la solution de l'équation homogène

\[y'(x) = - \dfrac{1}{x} y(x), \quad x\in \mathbb{R}_+^*.\]

Et les termes restants sont associés à une solution particulière \(y_p(x) = \arctan(x) - \dfrac{1}{2x} \ln(x^2+1), x\in \mathbb{R}_+^*,\) de l'équation différentielle

\[y'(x) = - \dfrac{1}{x} y(x) + f(x), \quad x\in \mathbb{R}_+^*,\]

avec

\[\forall x\in \mathbb{R}_+^*, \quad f(x) = y'_p(x) + \dfrac{1}{x} y_p(x) = \dfrac{1}{1+x^2} + \dfrac{1}{2x^2} \ln(x^2+1) - \dfrac{1}{2x} \dfrac{2x}{x^2+1} + \dfrac{1}{x} \arctan(x) - \dfrac{1}{2x^2} \ln(x^2+1) = \dfrac{1}{x} \arctan(x).\]

Par conséquent l'équation différentielle dont les solutions sont les fonctions précédentes est

\[x y'(x) = - y(x) + \arctan(x), \quad x\in \mathbb{R}_+^*.\]

1.b. Résoudre l'équation différentielle

\[y' + 2xy = -x.\]

Puis déterminer l'unique solution s'annulant en \(0\).

Correction

L'équation homogène associée est

\[y'(x) + 2xy = 0, \quad x\in \mathbb{R},\]

de solutions

\[x \in \mathbb{R} \longmapsto \lambda e^{-x^2}, \quad \lambda \in \mathbb{R}.\]

Et une solution particulière de l'équation différentielle est à chercher de la forme du second membre

\[y_p(x) = ax + b, \quad x\in \mathbb{R}, \quad a, b\in \mathbb{R}.\]

Alors la fonction \(y_p\) est solution de l'équation différentielle si et seulement si

\[\forall x\in \mathbb{R}, \quad a + 2ax^2 + 2bx = - x\]

i.e. si et seulement si

\[a = 0 \quad \text{et} \quad b = - \dfrac{1}{2}.\]

Par conséquent les solutions de l'équations sont données par

\[y(x) = \lambda e^{-x^2} - \dfrac{1}{2}, \quad x\in \mathbb{R}, \quad \lambda \in \mathbb{R}.\]

Puis la solution s'annulant en \(0\) vérifie

\[0 = y(0) = \lambda - \dfrac{1}{2}\]

i.e.

\[\lambda = \dfrac{1}{2}.\]

Donc la solution de l'équation différentielle s'annulant en \(0\) est donnée par

\[y(x) = \dfrac{1}{2} (e^{-x^2} - 1), \quad x\in \mathbb{R}.\]

2.a. Résoudre le problème de Cauchy

\[\begin{cases} y'' + 3y' + 2y = (3x^2 - 5)e^{-x} - 2xe^{-2x} \\ y(0) = y'(0) = 0. \end{cases}\]
Correction

L'équation caractéristique de l'équation homogène associée est donnée par

\[r^2 + 3r + 2 = 0\]

de discriminant \(\Delta = 9 - 8 = 1\) donc de solutions

\[r_1 = - 1 \quad \text{et} \quad r_2 = -2.\]

Ainsi les solutions de l'équation homogène sont données par

\[y(x) = \lambda e^{-x} + \mu e^{-2x}, \quad x\in \mathbb{R}, \quad \lambda,\mu \in \mathbb{R}.\]

Puis on cherche une solution de l'équation différentielle

\[y'' + 3y' + 2y = (3x^2 - 5) e^{-x}, \quad x\in \mathbb{R}.\]

Or \(-1\) est racine de l'équation caractéristique donc on cherche une solution particulière de la forme \(y_p(x) = (ax^3 + bx^2 + cx + d)e^{-x}, x\in \mathbb{R}\). Alors la fonction \(y_p\) est solution de l'équation différentielle ci-dessus si et seulement si

\[([6ax+ 2b - 6 ax^2 - 4bx - 2c + ax^3 + bx^2 + cx + d] + [9 ax^2 + 6 bx + 3c - 3ax^3 - 3bx^2 - 3cx - 3d] + [2ax^3 + 2bx^2 + 2cx + 2d])e^{-x} = (3x^2 - 5) e^{-x}, \quad x\in \mathbb{R},\]

i.e.

\[3ax^2 + (6a+2b)x + 2b + c = 3x^2 - 5, \quad x\in \mathbb{R},\]

i.e.

\[a = 1, \quad b = - 3a = -3, \quad c = -5-2b = 1, \quad d\in \mathbb{R}.\]

Donc une solution particulière est donnée par

\[y_1(x) = \left(x^3 - 3x^2 + x\right) e^{-x}, \quad x\in \mathbb{R}.\]

Puis on cherche une solution de l'équation différentielle

\[y'' + 3y' + 2y = -2xe^{-2x}, \quad x\in \mathbb{R}.\]

Or \(-2\) est racine de l'équation caractéristique donc on cherche une solution particulière de la forme \(y_p(x) = (ax^2 + bx +c)e^{-2x}, x\in \mathbb{R}\). Alors la fonction \(y_p\) est solution de l'équation différentielle ci-dessus si et seulement si

\[([2a - 8ax - 4b + 4ax^2 + 4bx + 4c] + [6ax + 3b - 6ax^2 - 6bx - 6c] + [2ax^2 + 2bx + 2c])e^{-2x} = -2xe^{-2x}, \quad x\in \mathbb{R},\]

i.e.

\[-2ax + 2a - b = -2x, \quad x\in \mathbb{R},\]

i.e.

\[a = 1, \quad b = 2, \quad c\in \mathbb{R}.\]

Donc une solution particulière est donnée par

\[y_2(x) = (x^2+2x)e^{-2x}, \quad x\in \mathbb{R}.\]

Par conséquent, d'après le principe de superposition, une solution particulière de l'équation différentielle de l'énoncé est donnée par

\[y_p(x) = y_1(x) + y_2(x) = (x^3 - 3x^2 + x) e^{-x} + (x^2+2x)e^{-2x}, \quad x\in \mathbb{R}.\]

Puis les solutions de l'équation sont données par

\[y(x) = \lambda e^{-x} + \mu e^{-2x} + (x^3 - 3x^2 + x) e^{-x} + (x^2+2x)e^{-2x}, \quad x\in \mathbb{R}, \quad \lambda \in \mathbb{R}.\]

Enfin la solution vérifiant \(y(0) = y'(0) = 0\) vérifie

\[0 = \lambda + \mu \quad \text{et} \quad 0 = -\lambda - 2\mu +1+2 = -\lambda -2\mu +3.\]

Donc, après résolution,

\[\lambda = -3\quad \text{et} \quad \mu = 3.\]

Par conséquent la solution est donnée par

\[y(x) = (x^3 - 3x^2- 5x - 3) e^{-x} + (-x^2+2x + 3)e^{-2x}, \quad x\in \mathbb{R}.\]

2.b. Résoudre le problème de Cauchy dans \(\mathbb{R}\)

\[\begin{cases} y'' - 2y' + 5y = e^x \sin(2x) \\ y(0) = y'(0) = 0. \end{cases}\]
Correction

L'équation caractéristique de l'équation homogène associée est

\[r^2 - 2r + 5 = 0\]

de discriminant \(\Delta = -16 < 0\) donc de solutions

\[x_1 = 1+2i \quad \text{et} \quad x_2 = 1-2i.\]

Donc les solutions réelles de l'équation homogènes sont données par

\[y(x) = \left(\lambda \cos(2x) + \mu \sin(2x) \right)e^x, \quad x\in \mathbb{R}, \quad \lambda, \mu \in \mathbb{R}.\]

On considère l'équation différentielle complexe associée

\[y'' - 2 y' + 5y = e^{(1+2i)x}, \quad x\in \mathbb{R}.\]

Or \(1+2i\) est solution de l'équation caractéristique donc on cherche une solution particulière de la forme \(y_p(x) = (ax+b)e^{(1+2i)x}, x\in \mathbb{R}\). Alors la fonction \(y_p\) est solution de l'équation différentielle complexe si et seulement si

\[(2(1+2i)a + (1+2i)^2 (ax+b) - 2 a - 2(1+2i)(ax+b) + 5(ax+b))e^{(1+2i)x} = e^{(2+i)x}, \quad x\in \mathbb{R},\]

i.e.

\[4ia = 1, \quad x\in \mathbb{R},\]

i.e.

\[a = \dfrac{1}{4i} = - \dfrac{i}{4} \quad \text{et} \quad b \in \mathbb{R}.\]

Donc une solution particulière de l'équation différentielle complexe est donnée par

\[y_p(x) = - \dfrac{ix}{4} e^{(1+2i)x}, \quad x\in \mathbb{R}.\]

Par conséquent on obtient une solution particulière de l'équation différentielle de l'énoncé en prenant la partie imaginaire

\[y_p(x) = - \dfrac{x}{4} \cos(2x) e^{x}, \quad x\in \mathbb{R}.\]

Par conséquent les solutions de l'équation différentielle de l'énoncé sont de la forme

\[y(x) = \left(\lambda \cos(2x) + \mu \sin(2x) - \dfrac{x}{4} \cos(2x) \right)e^x, \quad x\in \mathbb{R}, \quad \lambda,\mu \in \mathbb{R}.\]

Puis si \(y(0) = y'(0) = 0\) alors

\[0 = \lambda \quad \text{et} \quad 0 = 2 \mu - \dfrac{1}{4}\]

i.e.

\[\lambda = 0 \quad \text{et} \quad \mu = \dfrac{1}{8}.\]

Par conséquent

\[y(x) = \left(\dfrac{1}{8} \sin(2x) - \dfrac{x}{4} \cos(2x)\right) e^x, \quad x\in \mathbb{R}.\]

Exercice 9 : Nombres réels

1. On considère l'ensemble \(\mathbb{R}^2\) muni de l'ordre lexicographique \(\leq\) :

\[\forall (x_1,y_1), (x_2, y_2) \in \mathbb{R}^2, \quad (x_1,y_1) \leq (x_2, y_2) \quad \Longleftrightarrow \quad [x_1 < x_2 \quad \text{ou} \quad (x_1 = x_2 \quad \text{et} \quad y_1 \leq y_2)].\]

On considère également la partie \(A\) de \(\mathbb{R}^2\) définie par

\[A = \{(0,y), \quad y\in \mathbb{R}\}.\]

Montrer que la partie \(A\) est majoré dans \(\mathbb{R}^2\) mais que la partie \(A\) n'admet pas de plus petit majorant dans \(\mathbb{R}^2\).

Correction

Soit \((0,y) \in A\). Alors \((0,y) \leq (1,0)\) car \(0\leq 1\). Donc la partie \(A\) est majorée par \((1,0)\) par exemple. On suppose par l'absurde que la partie \(A\) admette un plus petit majorant \((x_0,y_0) \in \mathbb{R}^2\) :

\[\forall y \in \mathbb{R}, \quad (0,y) \leq (x_0,y_0)\]

et

\[\forall (x_1,y_1) \in \mathbb{R}^2, \quad [\forall y \in \mathbb{R}, \quad (0,y) \leq (x_1,y_1)] \quad \Longrightarrow \quad (x_0,y_0) \leq (x_1,y_1).\]

En particulier \((0,0) \leq (x_0,0)\) d'où \(x_0 \geq 0\). Soit \(n\in \mathbb{N}^*\) et \((x_n,y_n) = \left( \dfrac{1}{n}, y_0 \right)\). Alors

\[\forall y\in \mathbb{R}, \quad (0,y) \leq \left( \dfrac{1}{n}, y_0 \right).\]

Donc

\[(x_0,y_0) \leq \left( \dfrac{1}{n}, y_0 \right).\]

Ainsi \(0\leq x_0 \leq \dfrac{1}{n}\), d'où en faisant tendre \(n\) vers \(+\infty\), \(x_0 = 0\) et \((x_0,y_0) = (0,y_0)\). Sauf que \((0,y_0)\) n'est pas un majorant de la partie \(A\) :

\[(0,y_0) \leq (0,y_0+1) \in A.\]

Nous sommes donc arrivés à une contradiction : la partie \(A\) n'admet pas de plus petit majorant.

2. On considère la partie \(A\) de \(\mathbb{Q}\) définie par

\[A = \{r\in \mathbb{Q}, \quad r^2 \leq 2\}.\]

Montrer que la partie \(A\) est majorée dans \(\mathbb{Q}\) mais que la partie \(A\) n'admet pas de plus petit majorant dans \(\mathbb{Q}\).

Correction

Soit \(r \in A\). Alors \(r^2 \leq 2 \leq 4\) d'où \(r \leq 2\). Ainsi la partie \(A\) est majorée par \(2\) par exemple. On suppose par l'absurde que la partie \(A\) admet un plus petit majorant dans \(\mathbb{Q}\) : il existe \(r_0 \in \mathbb{Q}\) tel que

\[\forall r\in A, \quad r\leq r_0\]

et

\[\forall r_1 \in \mathbb{Q}, \quad [\forall r\in A, \quad r\leq r_1] \quad \Longrightarrow \quad r_0 \leq r_1.\]

En particulier \(r_0 \geq 1 > 0\) car \(1 \in A\). Or la partie \(\mathbb{Q}\) est dense dans \(\mathbb{R}\) donc il existe \((r_n)_{n\in \mathbb{N}} \in \mathbb{Q}^\mathbb{N}\) tel que

\[r_n \geq \sqrt{2} \quad \text{et} \quad r_n \underset{n\to +\infty}{\longrightarrow} \sqrt{2}.\]

Soit \(n\in \mathbb{N}^*\). Alors \(r_n\) est un majorant de la partie \(A\) :

\[\forall r\in A, \quad r \leq \sqrt{2} \leq r_n.\]

Donc \(r_0 \leq r_n\). De plus \(r_0 \geq \sqrt{2}\) car si \(r_0 < \sqrt{2}\) alors par densité de \(\mathbb{Q}\) dans \(\mathbb{R}\) il existe \(r\in \mathbb{Q}\) tel que \(r_0 < r < \sqrt{2}\) donc \(r_0^2 < r^2 < 2\) ce qui contredit le fait que \(r_0\) est un majorant de la partie \(A\). Par conséquent nous avons

\[\sqrt{2} \leq r_0 \leq r_n.\]

Donc en faisant tendre \(n\) vers \(+\infty\) on obtient \(\sqrt{2} = r_0 \in \mathbb{Q}\) ce qui est absurde. Par conséquent la partie \(A\) n'admet pas de plus petit majorant dans \(\mathbb{Q}\).

Exercice 10 : Suites

On considère \(a,b,c,d\in \mathbb{R}\) tels que \(c(ad-bc) \neq 0\), et la fonction \(f\) définie sur \(\mathbb{R} \backslash \left\{- \dfrac{d}{c}\right\}\) par

\[\forall x\in \mathbb{R} \backslash \left\{-\dfrac{d}{c} \right\}, \quad f(x) = \dfrac{ax+b}{cx+d}.\]

1. Montrer que l'équation \(f(x) = x\) admet soit 1 ou 2 solutions dans \(\mathbb{R} \backslash \left\{ - \dfrac{d}{c} \right\}\) soit aucune solution réelle.

Correction

Soit \(x\in \mathbb{R} \backslash \left\{-\dfrac{d}{c} \right\}\). Alors \(f(x) = x\)

si et seulement si

\[ax+b = x(cx+d) = cx^2 + dx\]

i.e.

\[cx^2 + (d-a)x + b = 0.\]

Nous avons donc une équation polynomiale de degré 2 qui admet 0, 1 ou 2 solutions dans \(\mathbb{R}\), et comme \(-\dfrac{d}{c}\) n'est pas une solution, dans \(\mathbb{R} \backslash \left\{-\dfrac{d}{c} \right\}\).

2. On suppose que l'on peut considérer la suite \((u_n)_{n\in \mathbb{N}}\) définie par

\[u_0 \in \mathbb{R}\backslash \left\{ - \dfrac{d}{c} \right\} \quad \text{et} \quad \forall n\in \mathbb{N}, \quad u_{n+1} = f(u_n).\]

Montrer que si la fonction \(f\) n'a pas de points fixes \((f(x) = x)\) alors la suite \((u_n)_{n\in \mathbb{N}}\) diverge.

Correction

On suppose que la suite \((u_n)_{n\in \mathbb{N}}\) converge vers \(\ell \in \mathbb{R}\). Alors par caractérisation séquentielle de la continuité \(f(\ell) = \ell\) donc la fonction \(f\) admet au moins un point fixe. On en déduit le résultat souhaité par contraposée.

3. On suppose que la fonction \(f\) admet au moins un point fixe \(\alpha\). Montrer que s'il existe un entier \(p \in \mathbb{N}\) tel que \(u_p = \alpha\) alors la suite \((u_n)_{n\in \mathbb{N}}\) est stationnaire puis que si \(p>0\) alors \(u_{p-1} = \alpha\) et en déduire que la suite \((u_n)_{n\in \mathbb{N}}\) est constante.

Correction
  • On suppose qu'il existe \(p\in \mathbb{N}\) tel que \(u_p = \alpha\). Alors on montre par récurrence que pour tout \(n\geq p\), \(u_n = \alpha\).

  • On suppose de plus que \(p > 0\). Or la fonction \(f\) est inversible de fonction inverse \(f^{-1}\) donnée par

\[\forall y \in \mathbb{R}\backslash \left\{ \dfrac{a}{c} \right\}, \quad f^{-1}(y) = -\dfrac{dy+b}{cy-a}.\]

Ainsi, comme \(f(u_p) = u_p\) et \(f(u_{p-1}) = u_p\), on en déduit, par injectivité, que \(u_{p-1} = u_p = \alpha\).

  • Par conséquent, par itérations successives ou récurrence, \(u_n = \alpha\) pour tout \(n\in \{0, ..., p-1\}\). Donc \((u_n)_{n\in \mathbb{N}}\) est la suite constante égale à \(\alpha\).

4. On suppose que la fonction \(f\) admet deux points fixes distincts \(\alpha\) et \(\beta\) et que \(u_0 \neq \alpha\). On considère alors la suite \((v_n)_{n\in \mathbb{N}}\) définie par

\[\forall n\in \mathbb{N}, \quad v_n = \dfrac{u_n - \beta}{u_n - \alpha}.\]

4.a. Montrer que la suite \((v_n)_{n\in \mathbb{N}}\) est géométrique de raison \(\dfrac{c\alpha + d}{c\beta + d}\).

Correction

Soit \(n\in \mathbb{N}\). Alors

\[u_{n+1} - \beta = \dfrac{au_n + b}{cu_n + d} - \dfrac{a\beta + b}{c\beta + d} = \dfrac{(au_n + b)(c\beta + d) - (a\beta + b)(cu_n+d)}{(cu_n + d)(c\beta +d)} = \dfrac{au_n d + bc\beta- a\beta d - bcu_n}{(cu_n + d)(c\beta +d)} = \dfrac{(ad - bc)(u_n - \beta)}{(cu_n + d)(c\beta +d)}.\]

De même

\[u_{n+1} - \alpha = \dfrac{(ad - bc)(u_n - \alpha)}{(cu_n + d)(c\alpha +d)}.\]

Donc

\[v_{n+1} = \dfrac{u_{n+1} - \beta}{u_{n+1} - \alpha} = \dfrac{u_n - \beta}{u_n - \alpha} \dfrac{c\alpha + d}{c\beta + d} = v_n \dfrac{c\alpha + d}{c\beta + d}.\]

4.b. Exprimer alors, pour \(n\in \mathbb{N}\), \(v_n\) en fonction de \(n\) et \(u_0\), puis \(u_n\) en fonction de \(n\) et \(u_0\).

Correction
  • Nous avons donc, pour \(n\in \mathbb{N}\), en notant \(k = dfrac{c\alpha + d}{c\beta + d}\)
\[v_n = \left( \dfrac{c\alpha + d}{c\beta + d} \right)^n v_0 = k^n \dfrac{u_0 - \beta}{u_0 - \alpha}.\]
  • Or
\[v_n(u_n - \alpha) = u_n - \beta\]

i.e.

\[u_n = \dfrac{\alpha v_n - \beta}{v_n-1} = \dfrac{\alpha k^n(u_0-\beta) - \beta(u_0-\alpha)}{\alpha k^n (u_0 - \beta) - (u_0 - \alpha)}.\]

4.c. En déduire une condition sur \(u_0\) pour que la suite \((u_n)_{n\in \mathbb{N}}\) soit bien définie. (Une condition du type \(u_0 \notin E\)\(E\) l'ensemble des termes d'une certaine suite.)

Correction

Le terme \(u_n\) est bien défini tant que \(u_n \neq - \dfrac{d}{c}\). Or \(u_n = - \dfrac{d}{c}\) si et seulement si

\[v_n = \dfrac{u_n - \beta}{u_n - \alpha} = \dfrac{c\beta + d}{c\alpha + d} = \dfrac{1}{k}\]

si et seulement si, comme \(v_n = k^n v_0\),

\[v_0 = \dfrac{1}{k^{n+1}}\]

si et seulement si

\[u_0 = \dfrac{\alpha - \beta k^{n+1}}{1-k^{n+1}}.\]

Au passage bien défini car \(\alpha \neq \beta\) i.e. \(k\neq 1\). Donc la suite \(u\) est bien définie si et seulement si

\[u_0 \notin \left\{ \dfrac{\alpha - \beta k^{n+1}}{1-k^{n+1}}, \quad n\in \mathbb{N} \right\}.\]

5. On suppose que la fonction \(f\) admet un unique point fixe \(\alpha\) et que \(u_0 \neq \alpha\). On considère alors la suite \((v_n)_{n\in \mathbb{N}}\) définie par

\[\forall n\in \mathbb{N}, \quad v_n = \dfrac{1}{u_n - \alpha}.\]

5.a. Montrer que la suite \((v_n)_{n\in \mathbb{N}}\) est arithmétique.

Correction

Soit \(n\in \mathbb{N}\). Alors

\[v_{n+1} = \dfrac{1}{u_{n+1} - \alpha} = \dfrac{(cu_n+d)(c\alpha + d)}{(au_n+b)(c\alpha + d) - (a\alpha + b)(cu_n + d)} = \dfrac{(cu_n+d)(c\alpha + d)}{adu_n + bc\alpha - a\alpha d - bcu_n} = \dfrac{(cu_n+d)(c\alpha + d)}{(ad - bc) (u_n - \alpha)}.\]

Or la fonction \(f\) admet un unique point fixe donc d'après la question 1. l'équation polynomiale \(cx^2 + (d-a)x +b = 0\) admet un discriminant nul

\[0 = (d-a)^2 - 4bc\]

et pour solution

\[\alpha = - \dfrac{d-a}{2c} = \dfrac{a-d}{2c}.\]

En particulier, \(b\) étant le terme constant de cette équation,

\[b = c \alpha^2.\]

Donc

\[c\alpha + d = \dfrac{a-d}{2} + d = \dfrac{a+d}{2} \quad \text{et} \quad ad - bc = ad - c^2 \alpha^2 = ad - \dfrac{(a-d)^2}{4} = \dfrac{4ad - (a-d)^2}{4} = \dfrac{(a+d)^2}{4}\]

5.b. Exprimer alors, pour \(n\in \mathbb{N}\), \(v_n\) en fonction de \(n\) et \(u_0\), puis \(u_n\) en fonction de \(n\) et \(u_0\).

Correction

5.c. En déduire une condition sur \(u_0\) pour que la suite \((u_n)_{n\in \mathbb{N}}\) soit bien définie. (Une condition du type \(u_0 \notin E\)\(E\) l'ensemble des termes d'une certaine suite.)

Correction

Exercice 11 : Limites

Etudier les limites des fonctions suivantes au point indiqué.

1. \(x \sin\left( \dfrac{1}{x} \right)\) en 0

Correction

2. \(\dfrac{x \cos(e^x)}{x^2 + 1}\) en \(+\infty\)

Correction

3. \(e^{x-\sin(x)}\) en \(+\infty\)

Correction

4. \(\dfrac{x+\arctan(x)}{x}\) en \(+\infty\)

Correction

5. \(x \left\lfloor \dfrac{1}{x} \right\rfloor\) en \(0\)

Correction

6. \(x \left\lfloor \dfrac{1}{x} \right\rfloor\) en \(+\infty\)

Correction

7. \(\left\lfloor \dfrac{1}{x} \right\rfloor\) en \(0\)

Correction

8. \(x^2\left\lfloor \dfrac{1}{x} \right\rfloor\) en \(0\)

Correction

Exercice 12 : Continuité

1. Soit \(f : \mathbb{R} \longrightarrow \mathbb{R}\) continue telle que

\[\forall x,y\in \mathbb{R}, \quad f(x+y) = f(x) + f(y).\]

Montrer qu'il existe \(\alpha \in \mathbb{R}\) tel que

\[\forall x\in \mathbb{R}, \quad f(x) = \alpha x.\]
Correction

2. Soit \(f : \mathbb{R} \longrightarrow \mathbb{R}\) continue telle que

\[\forall (x,y) \in \mathbb{R}^2, \quad f(x+y) = f(x) f(y).\]

2.a. Montrer que \(f\geq 0\).

Correction

2.b. Montrer que la fonction \(f\) est soit la fonction nulle soit une fonction ne s'annulant jamais.

Correction

2.c. Dans le second cas, déterminer une équation fonctionnelle vérifiée par la fonction \(g = \ln \circ f\).

Correction

2.d. Déterminer alors toutes les fonctions \(f\) continues sur \(\mathbb{R}\) telles que

\[\forall (x,y) \in \mathbb{R}^2, \quad f(x+y) = f(x) f(y).\]
Correction

Exercice 13 : Dérivation

On considère la fonction \(f\) définie par

\[f(x) = \dfrac{\ln(1+x)}{x}.\]

1. Déterminer l'ensemble de définition \(D\) de la fonction \(f\).

Correction

2 Montrer que la fonction \(f\) admet en \(0\) un prolongement par continuité. Par quelle valeur ? On notera \(D'\) l'ensemble de définition du prolongement que l'on notera encore \(f\).

Correction

3. La fonction \(f\) est-elle dérivable en \(0\) ? Si oui déterminer \(f'(0)\).

Correction

4. Calculer la dérivée de la fonction \(f\) sur \(D\) puis montrer que la fonction \(f\) est de classe \(C^1\) sur \(D'\).

Correction

5. Etudier les variations de la fonction \(f\) et dresser son tableau de variation.

Correction

6. Montrer que la fonction \(f\) est de classe \(C^\infty\) sur \(D\).

Correction

7. Calculer la dérivée seconde de la fonction \(f\) sur \(D\).

Correction

8. Montrer que pour tout \(n\in \mathbb{N}^*\) il existe \(T_n \in \mathbb{R}[X]\) et \(a_n \in \mathbb{R}\) tels que

\[\forall x\in D, \quad f^{(n)}(x) = \dfrac{T_n(x)}{(1+x)^n x^n} + a_n \dfrac{\ln(1+x)}{x^{n+1}}.\]
Correction

9. Montrer que tous les cœfficients du polynôme \(T_n\) sont des entiers.

Correction

10. En utilisant la formule de Leibniz calculer la dérivée \(n\)-ième \(f^{(n)}\) et en déduire la valeur du polynôme \(T_n\). (On ne cherchera pas à expliciter une expression de chacun des cœfficients de ce polynôme.)

??? success "Correction"Vérifier cette expression pour \(n = 2\).

Exercice 14 : Structures algébriques

Pour tout \(x \in \mathbb{Z}\) et \(n\in \mathbb{N}\) tel que \(n \geq 2\), on note \(\overline{x}\) la classe d'équivalence de \(x\) pour la relation de congruence modulo \(n\) :

\[\overline{x} = \{y\in \mathbb{Z}, \quad y \equiv x [n]\}.\]

On parlera abusivement de \(\overline{x}\) comme étant un élément de \(\overline{x}\). Autrement dit \(\overline{x}\) est n'importe quel représentant de la classe d'équivalence de \(x\).

1. Soit \(x,x' \in \mathbb{Z}\). Montrer que si \(x\equiv x'[n]\) alors \(\overline{x} = \overline{x'}\). En déduire que l'ensemble \(\{\overline{x}, x\in \mathbb{Z}\}\) est fini de cardinal \(n\). On le note \(\mathbb{Z}/n\mathbb{Z}\).

Correction

2. Montrer que \(\mathbb{Z}/n\mathbb{Z}\) est un anneau pour les lois

\[\overline{x} + \overline{y} = \overline{x+y} \quad \text{et} \quad \overline{x} ~ \overline{y} = \overline{xy}.\]
Correction

3. Montrer que si \(n\) n'est pas premier alors l'anneau \(\mathbb{Z}/n\mathbb{Z}\) n'est pas intègre.

Correction

4. Montrer que l'anneau \(\mathbb{Z} / n\mathbb{Z}\) est un corps si et seulement si \(n\) est premier.

Correction

5. On considère \(p\in \mathcal{P}\) impair. On dit que \(a \in \mathbb{Z}\) est un carré modulo \(p\) s'il existe \(x\in \mathbb{Z}\) tel que

\[a \equiv x^2[p].\]

5.a. Montrer que dans \(\mathbb{Z}/p\mathbb{Z}\) il y a \(\dfrac{p+1}{2}\) carrés. On pourra utiliser le morphisme carré sur \((\mathbb{Z}/p\mathbb{Z})^*\) pour montrer qu'il y a autant de carrés que de non carrés dans \((\mathbb{Z}/p\mathbb{Z})^*\).

Correction

5.b. Soit \(x\in \mathbb{Z}\) tel que \(\overline{x} \neq \overline{0}\). Montrer que \(\overline{x}\) est un carré modulo \(p\) si et seulement si \(x^{\frac{p-1}{2}} \equiv 1[p]\).

Correction

5.c. Montrer que \(-1\) est un carré modulo \(p\) si et seulement si \(p\equiv 1[4]\).

Correction

Exercice 15 : Arithmétique

Pour tout \(n\in \mathbb{N}^*\) on considère \(\sigma(n)\) la somme des diviseurs positifs de \(n\).

1. Calculer \(\sigma(6), \sigma(7)\) et \(\sigma(8)\).

Correction

2. Montrer que

\[\forall n\in \mathbb{N}\backslash\{0,1\}, \quad \sigma(n) \geq n+1.\]

A quelle condition nécessaire et suffisante a-t-on égalité ?

Correction

3.a. Soient \(p,q \in \mathcal{P}\) distincts et \(n = p q\). Montrer que

\[\sigma(n) = \sigma(p) \sigma(q).\]
Correction

3.b. On considère l'assertion suivante.

\[P : \quad \forall m,n\in \mathbb{N}^*, \quad m\neq n \quad \Longrightarrow \quad \sigma(mn) = \sigma(m) \sigma(n).\]

Cette assertion est-elle vraie ?

Correction

4. Montrer que pour tout \(p\in \mathcal{P}\) et \(k\in \mathbb{N}\)

\[\sigma(p^k) = \dfrac{p^{k+1} - 1}{p-1}.\]
Correction

5. Soient \(p,q\in \mathcal{P}\) distincts, \(k,\ell \in \mathbb{N}\) et \(n = p^k q^\ell\).

5.a. Soit \(d\in \mathbb{N}^*\). Montrer que \(d\mid n\) si et seulement s'il existe \(i,j\in \{0, ..., k\} \times \{0, ..., \ell\}\) tels que \(d = p^i q^j\).

Correction

5.b. En déduire l'égalité

\[\sigma(n) = \sigma(p^kq^\ell) = \dfrac{p^{k+1} - 1}{p-1} \dfrac{q^{\ell +1} - 1}{q-1}.\]
Correction

Exercice 16 : Espaces vectoriels

On considère l'espace vectoriel \(E = C^\infty(]-1,1[,\mathbb{R})\) et la partie \(F\) l'ensemble des fonctions polynomiales réelles restreintes au départ à \(]-1,1[\).

1. Montrer que la partie \(F\) est un sous-espace vectoriel de l'espace \(E\).

Correction

2. On considère l'application \(\varphi\) définie sur \(E\) par

\[\forall f\in E, \quad \forall x\in \mathbb{E}, \quad \varphi(f)(x) = (x^2-1) f''(x) + 2 x f'(x).\]

2.a. Montrer que l'application \(\varphi\) est un endomorphisme de l'espace \(E\).

Correction

2.b. Déterminer son noyau \(\text{ker}(\varphi)\) et \(\text{ker}(\varphi) \cap F\).

Correction

2.c. Montrer que le sous-espace vectoriel \(F\) est stable par l'endormorphisme \(\varphi\) : \(\varphi(F) \subset F\).

Correction

2.d. Soit \(P \in F\) de degré \(n\in \mathbb{N}\). Calculer le terme de plus haut degré de la fonction polynomiale \(\varphi(P)\).

Correction

2.e. Déterminer une fonction polynomiale \(K \in F\) tel que \(K(0) = 1\) et \(\varphi(K) = 6K\).

Correction

2.f. Exprimer en fonction de \(K\) les fonctions polynomiales \(P\) vérifiant \(\varphi(P) = 6P\).

Correction

Exercice 17 : Familles de vecteurs

1. Soient \(n \in \mathbb{N}^*, \alpha_1, ..., \alpha_n \in \mathbb{C}\) distincts, \(A= \text{diag}(\alpha_1, ..., \alpha_n)\) matrice diagonale de cœfficients diagonaux \(\alpha_1, ..., \alpha_n\) et

\[C(A) = \{M\in \mathcal{M}_n(\mathbb{C}), \quad AM = MA\}.\]

1.a. Montrer que la partie \(C(A)\), appelé commutant de la matrice \(A\), est un sous-espace vectoriel de \(\mathcal{M}_n(\mathbb{C})\).

Correction

1.b. Montrer que \((A^k)_{0\leq k\leq n-1}\) est une base du sous-espace \(C(A)\).

Correction

2. On considère les fonctions suivantes définies pour \(x \in ~]-1,1[\)

\[f_1(x) = \sqrt{\dfrac{1+x}{1-x}}, \quad f_2(x) = \sqrt{\dfrac{1-x}{1+x}}, \quad f_3(x) = \dfrac{1}{\sqrt{1-x^2}}, \quad f_4(x) = \dfrac{x}{\sqrt{1-x^2}}.\]

Quel est le rang de la famille \((f_1, f_2, f_3, f_4)\) ?

Correction

Exercice 18 : Applications linéaires

On considère l'espace vectoriel \(E = \mathbb{R}^4\) et un endomorphisme \(u\) de l'espace \(E\) tel que

\[u^2 + u + \text{id}_E = 0.\]

1.a. Montrer que pour tout \(x\in E\backslash \{0\}\) la famille \((x,u(x))\) est une famille libre.

Correction

Soit \(\lambda, \mu \in \mathbb{R}\) tel que

\[\lambda x + \mu u(x) = 0.\]

Donc, en appliquant l'application linéaire \(\mu u\),

\[\lambda \mu u(x) + \mu^2 u^2(x) = 0\]

i.e., comme \(\mu u(x) = - \lambda x\) et \(\mu u^2(x) = -\mu u(x) - \mu x = (\lambda - \mu) x\),

\[(-\lambda^2 + \mu \lambda - \mu^2) x = 0.\]

Donc, comme \(x\neq 0\),

\[\lambda^2 - \mu \lambda + \mu^2 = 0.\]

Or le polynôme \(X^2 - \mu X + \mu^2\) admet comme discriminant \(\Delta = \mu^2 - 4 \mu^2 = - 3 \mu^2 \leq 0\). Donc si \(\mu \neq 0\) alors \(\lambda\) est une racine non réelle ce qui ne peut pas donc \(\mu = 0\) et \(\lambda = \dfrac{\mu}{2} = 0\).

1.b. Montrer que l'endomorphisme \(u\) est un automorphisme de l'espace \(E\) et déterminer \(u^{-1}\).

Correction

Nous avons

\[u(-u-\text{id}_E) = -u^2 - u = \text{id}_E.\]

Donc l'endomorphisme est inversible d'inverse

\[u^{-1} = -u-\text{id}_E.\]

2. Soient \(x,y \in E\) tels que la famille \((x,y,u(x))\) soit libre. Montrer que la famille \((x,y,u(x),u(y))\) est une famille libre.

Correction

Soit \(a,b,c,d \in \mathbb{R}\) tels que

\[(1) \quad ax + by + cu(x) + du(y) = 0.\]

Alors

\[au(x) + bu(y) + cu^2(x) + du^2(y) = 0.\]

Or \(u^2(x) = -u(x) - x\) et \(u^2(y) = -u(y) -y\) donc

\[(2) \quad -cx -dy + (a-c) u(x) + (b-d) u(y) = 0.\]

Donc en effectuant l'opération \((b-d)(1) - d(2)\) nous obtenons

\[((b-d)a+dc)x + ((b-d)b + d^2)y + ((b-d)c - d(a-c)) u(x) = 0.\]

Or la famille \((x,y,u(x))\) est libre, donc

\[(b-d) a + dc = (b-d)b+d^2 = (b-d)c - d(a-c) = 0.\]

Donc, grâce au second terme et au raisonnement de la question 1.a., nous obtenons \(b = d = 0\) et ainsi l'équation \((1)\) devient

\[ax + cu(x) = 0.\]

Or d'après la question 1.a., la famille \((x,u(x))\) est libre, d'où \(a = c = 0\).

3. Montrer qu'il existe une base \(\mathcal{B} = (e_1, e_2,e_3,e_4)\) de l'espace \(E\) telle que \(e_3 = u(e_1)\) et \(e_4 = u(e_2)\).

Correction

Comme \(u^2 + u + \text{id}_E = 0\), nous avons \(u\neq 0\), donc il existe \(x\in E\) tel que \(u(x) \neq 0\). Donc d'après la question 1.a. la famille \((x,u(x))\) est libre. Puis, d'après le théorème de la base incomplète, il existe deux vecteurs \(y,z\in E\) telle que la famille \((x,u(x),y,z)\) soit une base de l'espace \(E\). Donc en particulier la famille \((x,y,u(x))\) est libre. Donc, d'après la question 2., la famille \((x,y,u(x),u(y))\) est une famille libre de \(4 = \text{dim}(E)\) vecteurs de l'espace \(E\). Donc il s'agit d'une base \((e_1, e_2, e_3, e_4)\) avec \(e_3 = u(x) = u(e_1)\) et \(e_4 = u(y) = u(e_2)\).